Vous êtes sur la page 1sur 7

Tema 14

Continuidad y monotona

Generalizando lo que se hizo en su momento para sucesiones, definiremos la monotona de


una funcin, en forma bien fcil de adivinar. Probaremos entonces dos resultados importantes
que relacionan la continuidad de una funcin con su monotona. Como principal consecuencia,
deduciremos que la inversa de una funcin continua e inyectiva, definida en un intervalo, es
tambin una funcin continua.

14.1. Funciones montonas


Las definiciones que siguen son tan intuitivas que no precisan ninguna motivacin. Diremos
que una funcin f : A R es
creciente cuando: x, y A , x < y f (x) 6 f (y)
decreciente cuando: x, y A , x < y f (x) > f (y), es decir, cuando f es creciente
montona cuando es creciente o decreciente.
Puesto que las sucesiones de nmeros reales son funciones reales de variable real, conviene
observar que las definiciones anteriores generalizan claramente a las que dimos en su momento
para sucesiones.
Volviendo al caso general, obsrvese que una funcin f : A R es a la vez creciente y
decreciente si, y slo si, es constante. Tambin es claro que, suponiendo que el conjunto A tiene
al menos tres puntos, existen funciones f : A R que no son montonas. Pero la propiedad que
ms nos interesa es la monotona estricta, que se define como sigue. Una funcin f : A R es
estrictamente creciente cuando: x, y A , x < y f (x) < f (y)
estrictamente decreciente cuando: x, y A , x < y f (x) > f (y), es decir, cuando f
es estrictamente creciente
estrictamente montona cuando es estrictamente creciente o estrictamente decreciente.

109
14. Continuidad y monotona 110

Sin duda, las seis propiedades de monotona recin introducidas son bastante exigentes. Lo
ms frecuente es que el conjunto de definicin de una funcin pueda expresarse como unin
finita de subconjuntos, de forma que la restriccin de la funcin a cada uno de ellos s verifique
algn tipo de monotona. Por ello es til la siguiente nomenclatura:
Dada una funcin f : A R y un conjunto no vaco B A, diremos que f es creciente en B
cuando la restriccin f |B sea una funcin creciente, es decir, cuando

x, y B , x 6 y = f (x) 6 f (y)

Anlogo criterio se sigue para los otros cinco tipos de monotona.


Como ejemplo, veamos las funciones potencia de exponente natural. Ms concretamente,
para cada k N, consideremos la funcin fk : R R definida por

fk (x) = x k x R

Usando propiedades bien conocidas de las potencias, comprobamos fcilmente lo siguiente:


Si k es impar, la funcin fk es estrictamente creciente (en R).
Si por el contrario k es par, la funcin fk no es montona, pero es estrictamente creciente

en R+0 y estrictamente decreciente en R0 .

Otro ejemplo destacable es la funcin valor absoluto: es estrictamente decreciente en R


0 y
+
estrictamente creciente en R0 , luego no es montona.
Nuestro objetivo es relacionar la continuidad de una funcin con su monotona. Conviene
resaltar que al definir ambas propiedades, para un subconjunto del conjunto de definicin, no
hemos seguido exactamente el mismo criterio: decir que una funcin f : A R es continua en
un conjunto B A no es lo mismo que decir que f |B es continua.

14.2. De la continuidad a la monotona


Evidentemente, toda funcin estrictamente montona es inyectiva. Bajo ciertas condiciones,
vamos a conseguir el recproco, y ste es el primer resultado clave sobre funciones continuas e
inyectivas:

Teorema. Sea I un intervalo y f : I R una funcin continua e inyectiva. Entonces f es


estrictamente montona.

Demostracin. Basta evidentemente probar que f es montona, pues de la inyectividad se


deduce entonces la monotona estricta. Consideremos primero el caso en que el intervalo I es
cerrado y acotado: I = [a, b] con a, b R , a < b. Por ser f inyectiva sabemos que f (a) 6= f (b)
y supondremos de momento que f (a) < f (b).
El primer paso ser probar que

x [a, b] = f (a) 6 f (x) 6 f (b) (1)


14. Continuidad y monotona 111

En efecto, usando el teorema del valor intermedio veremos que no puede ser f (x) < f (a)
y tampoco f (x) > f (b). Concretamente, si fuese f (x) < f (a) aplicaramos el teorema a la
restriccin de f al intervalo [x, b] que es continua y toma en dicho intervalo los valores f (x) y
f (b), luego debe tomar tambin el valor intermedio f (a). Por tanto, existira z [x, b] tal que
f (z) = f (a), pero esto contradice la inyectividad de f , ya que a < z. Anlogamente, si fuese
f (x) > f (b) aplicaramos el teorema del valor intermedio a la restriccin de f al intervalo [a, x]
obteniendo un z [a, x] tal que f (z) = f (b), lo que contradice otra vez la inyectividad de f .
A partir de (1) obtenemos inmediatamente que f es creciente. Tomados x, y [a, b] con
x < y, aplicando (1) tenemos f (a) < f (y). Pero entonces podemos aplicar (1) a la restriccin
de f al intervalo [a, y], que es continua e inyectiva, con f (a) < f (y), obteniendo f (x) 6 f (y)
como queramos.
En el caso f (a) > f (b), el razonamiento anterior se aplica a la funcin f , continua e
inyectiva en [a, b], con f (a) < f (b). Obtenemos que f es creciente, luego f es decreciente.
Queda as demostrado el teorema en el caso de que el intervalo I sea cerrado y acotado.
Vamos ahora al caso general en que I es un intervalo no vaco arbitrario. Si f no fuese
montona, existiran x1 , y1 , x2 , y2 I verificando las siguientes desigualdades:
x1 < y1 , x2 < y2 , f (x1 ) > f (y1 ) , f (x2 ) < f (y2 )
Tomamos entonces a = mn {x1 , x2 } y b = max {y1 , y2 }. Al ser I un intervalo, tenemos [a, b] I,
lo que permite considerar la restriccin de f al intervalo [a, b], que es una funcin continua
e inyectiva en dicho intervalo. Puesto que x1 , y1 , x2 , y2 [a, b] dicha restriccin no puede ser
montona, lo cual es una flagrante contradiccin con lo demostrado en el caso de un intervalo
cerrado y acotado. 

Vamos a ver ahora que el teorema anterior, combinado con el teorema del valor intermedio,
nos permite frecuentemente determinar la imagen de una funcin.
Dados a, b R con a < b, para una funcin continua e inyectiva f : [a, b] R, queremos
determinar su imagen: J = f [a, b] . Sean = f (a), = f (b) y supongamos de momento que
< . Gracias al teorema del valor intermedio, sabemos que J es un intervalo, y es obvio que
, J, luego [, ] J. Pero adems, gracias al teorema anterior, sabemos que f es creciente,
luego tenemos = f (a) 6 f (x) 6 f (b) = , para todo x [a, b]. En resumen, J = [, ]. Por
supuesto, de haber sido > , f habra sido decreciente y habramos obtenido J = [, ]. La
principal ventaja del teorema anterior estriba en que la inyectividad de una funcin suele ser
ms fcil de comprobar que su monotona.
Como ejemplo concreto, consideremos la funcin f : R R definida por
2x
f (x) = x [0, 1]
1 + x2
que es continua, por ser una funcin racional. Al estudiar su posible inyectividad, para x, y R
tenemos
f (x) = f (y) 2x(1 + y2 ) = 2y(1 + x2 ) (1 xy)(x y) = 0
luego f (x) = f (y) si, y slo si, x = y o xy = 1. Deducimos, por ejemplo, que las restricciones de
f a los intervalos ] , 1], [1, 1] y [1, +[ son inyectivas, luego estrictamente montonas.
14. Continuidad y monotona 112


Por ser 1 = f (1) < f (1) = 1, f es creciente en [1, 1], luego f ([1, 1] = [1, 1].
Adems f es decreciente,
 tanto en ] , 1]como en [1, [, de donde deducimos sin dificultad
que f ] , 1] = [1, 0[ y que f [1, +[ =]0, 1]. Concluimos que f (R) = [1, 1].

14.3. De la monotona a la continuidad


Aplicando la caracterizacin de la continuidad mediante sucesiones montonas, conseguimos
enseguida una til condicin suficiente para la continuidad de una funcin montona:

Teorema. Sea f : A R una funcin real de variable real. Supongamos que f es montona
y que f (A) es un intervalo. Entonces f es continua.

Demostracin. Podemos evidentemente suponer que f es creciente, pues en otro caso


bastara usar la funcin f , cuya imagen tambin es un intervalo. Fijado x A, para probar
que f es continua en el punto x, tomamos una sucesin montona {xn } de puntos de A tal que
{xn } x y bastar ver que { f (xn )} f (x).
Suponiendo primero que {xn } es creciente, para todo n N tenemos xn 6 xn+1 6 x, luego
f (xn ) 6 f (xn+1 ) 6 f (x), ya que f es creciente. Por tanto, la sucesin { f (xn )} es creciente y
mayorada, luego convergente. Poniendo L = lm { f (xn )} tenemos L 6 f (x) y, suponiendo que
L < f (x), llegaremos a contradiccin.
Tomando y R tal que L < y < f (x), tenemos claramente f (x1 ) < y < f (x) y, usando que
f (A) es un intervalo, deber existir a A tal que f (a) = y. Si fuese x 6 a el crecimiento de
f nos dara f (x) 6 f (a) = y cosa que no es cierta. Pero si fuese a < x, puesto que {xn } x,
podramos tomar m N tal que a < xm , con lo que y = f (a) 6 f (xm ) 6 L, cosa que tampoco es
cierta. Hemos demostrado que L = f (x), es decir, { f (xn )} f (x) como queramos.
Si la sucesin {xn } hubiese sido decreciente, un razonamiento enteramente anlogo nos
hubiera llevado a la misma conclusin. 

Conviene resaltar que, en el teorema anterior, el conjunto A no tiene por qu ser un intervalo.
Toda funcin montona, cuya imagen sea un intervalo, es continua.

14.4. Continuidad de la inversa


Para una funcin continua e inyectiva, es natural preguntarse si la funcin inversa tambin es
continua. Para tener una visin intuitiva del problema, observemos la relacin entre la grfica de
una funcin inyectiva f : A  R y la de su inversa, la funcin f 1 : f (A) R que se caracteriza
por verificar que f 1 f (x) = x para todo x A. Tenemos claramente
Gr f 1 = y, f 1 (y) : y f (A) = f (x), x : x A
   

Observamos que las grficas de f y f 1 se obtienen cada una a partir de la otra mediante
la simetra que tiene como eje la recta de ecuacin y = x, puesto que dicha simetra es la
transformacin (x, y) 7 (y, x), de R2 en s mismo.
14. Continuidad y monotona 113

Un sencillo dibujo ayuda a entender esta relacin entre las grficas de f y f 1 :

f 1

Con la interpretacin geomtrica comentada, se comprende muy bien el siguiente ejemplo


de una funcin continua e inyectiva cuya inversa no es continua. Consideremos el conjunto
A = [0, 1[{2} y la funcin f : A R definida por

f (2) = 1 y f (x) = x x [0, 1[

El carcter local de la continuidad nos permite comprobar fcilmente que f es continua. Por
otra parte, es claro que f es inyectiva, que f (A) = [0, 1] y f 1 : [0, 1] R viene dada por

f 1 (1) = 2 y f 1 (x) = x x [0, 1[

Como quiera que { f 1 (1 1/n)} = {1 1/n} 1 6= 2 = f 1 (1), observamos que f 1 no es


continua en el punto 1. Las grficas de f y f 1 permiten visualizar claramente que f es continua
mientras que f 1 no lo es:

1 1

1 2 1

Sin embargo, conviene observar que esta situacin ha sido posible porque el conjunto A
no es un intervalo. Para una funcin continua e inyectiva en un intervalo, la relacin entre las
grficas de la funcin y de su inversa permite intuir que dicha inversa debe ser continua, y eso
es lo que vamos a probar. Para ello usaremos los dos teoremas obtenidos anteriormente, junto
con una sencilla observacin:
14. Continuidad y monotona 114

Si f : A R es una funcin estrictamente creciente, entonces f 1 : f (A) R tambin


es estrictamente creciente. Si f es estrictamente decreciente, entonces f 1 tambin lo es.

La comprobacin de este hecho es inmediata. Supongamos que f es estrictamente creciente,


sean u, v f (A) con u < v y sean x = f 1 (u) , y = f 1 (v). Si fuese y 6 x, aplicando que f es
creciente tendramos v = f (y) 6 f (x) = u, que es una contradiccin, luego deber ser x < y, es
decir, f 1 (u) < f 1 (v). Esto prueba que f 1 es estrictamente creciente, como queramos. En
el caso de que f sea estrictamente decreciente, se razona de forma enteramente anloga. 
Resolvamos ya el problema planteado sobre la continuidad de la funcin inversa. En primer
lugar, tenemos:

Sea I un intervalo y sea f : I R una funcin estrictamente montona. Entonces f 1 es


continua.

En efecto, acabamos de ver que f 1 : f (I) R tambin es estrictamente montona, pero su


imagen es un intervalo, ya que f 1 f (I) = I, luego f 1 es continua. 
Como consecuencia inmediata tenemos:

Sea I un intervalo y f : I R una funcin continua e inyectiva. Entonces f 1 es continua.

En efecto: sabemos que f es estrictamente montona y basta aplicar el resultado anterior. 

Como ejemplo, obtenemos fcilmente la continuidad de la funcin raz k-sima. Dado k N,


consideramos la funcin fk : R+ +
0 R0 definida por

fk (x) = x k x R+
0
Desde que se prob la existencia de las races de nmeros positivos, sabemos que fk es biyectiva
y estrictamente creciente. Merece la pena comentar que ahora todo ello puede probarse de
forma mucho ms fcil. Por una parte, fk es una funcin continua e inyectiva en el intervalo
R+0 , luego es estrictamente montona, pero f (0) = 0 < 1 = f (1), luego f k es estrictamente
creciente. Por otra, su imagen es un intervalo J R+0 tal que 0 J y J no est mayorado, ya
k +
que { fk (n)} = {n } +, luego J = R0 .
Pero lo que ahora nos interesa es que la funcin inversa fk1 : R+ +
0 R0 , que evidentemente
viene dada por

fk1 (y) = k y y R+ 0
tambin es continua. Esto tambin era conocido, pues sabamos que si {yn } y con yn > 0

para todo n N, entonces { k yn } k y, pero de nuevo tenemos una demostracin ms rpida
y elegante.
A modo de repaso, concluimos este tema destacando en un solo enunciado la informacin
obtenida sobre una funcin continua en un intervalo:

Sea I un intervalo y f : I R una funcin continua. Entonces:


(i) f (I) es un intervalo.
(ii) Si I es cerrado y acotado, lo mismo le ocurre a f (I).
(iii) Si f es inyectiva, entonces f es estrictamente montona y f 1 es continua.
14. Continuidad y monotona 115

14.5. Ejercicios
1. Sea f : R R una funcin continua. Probar que si f |Q es montona, entonces f es
montona.

2. Sea I un intervalo y f : I R una funcin inyectiva. Analizar la relacin existente entre


las siguientes afirmaciones:
(i) f es continua
(ii) f (I) es un intervalo
(iii) f es estrictamente montona
(iv) f 1 es continua

3. Calcular la imagen de la funcin f : [1, 1] R definida por

x2
f (x) = x [1, 1]
1 + x2

4. Sea f : [1, 1] R la funcin definida por

2x
f (x) = x [1, 1]
1 + |x|

Probar que f es inyectiva y calcular su imagen.

5. Sea f :] 2, 2[ R la funcin definida por


x
f (x) = x ] 2, 2[
4 x2
   
Calcular f ] 2, 2[ , f [0, 2[ , f ] 1, 1[ y f [1, 1] .

6. Probar que, para cada y R+ 5 4


0 , la ecuacin x + x + x = y tiene una nica solucin
+
x R0 , y que denotando por g(y) a dicha solucin, se obtiene una funcin continua
g : R+ + 5 3
0 R0 . Deducir que si xn > 0 para todo n N y {xn + xn + xn } 3, entonces
{xn } 1.

Vous aimerez peut-être aussi